Đến nội dung

Hình ảnh

Việt Nam Team Selection Test 2012 - Đề bài, lời giải và danh sách đội tuyển


  • Please log in to reply
Chủ đề này có 22 trả lời

#1
Trần Đức Anh @@

Trần Đức Anh @@

    Thượng sĩ

  • Thành viên
  • 286 Bài viết
Đến hẹn lại lên,sau kì thi VMO 2011 diễn ra cách đây hơn 2 tháng thì vào ngày mai và ngày kia Bộ GD và ĐT sẽ tổ chức kì thi chọn đội tuyển VN tham dự kì thi Olympic Toán quốc tế năm 2012 diễn ra ở Argentina .Sáng nay , 42 thí sinh đến từ khắp miền Nam Bắc đã tụ họp ở ĐHSP Hà Nội nghe điểm danh và phổ biến những điều cần biết. Sáng mai sẽ tiến hành thi luôn, thời gian vẫn là 4h30' cho mỗi ngày, với 3 bài toán.
Mình mở topic này để mọi người post đề bài, thảo luận về kì thi này. Vậy, mai ai có đề thì gõ Latex sạch đẹp và viết lên #2 nhé, chú ý là #2 chỉ để post đề bài, vi phạm thì mình buộc phải xoá.
Chữ ký spam! Không cần xoá!

#2
hoangduc

hoangduc

    Trung sĩ

  • Thành viên
  • 108 Bài viết
Kỳ thi chọn đội tuyển Olympic 2012
Ngày thi thứ nhất
Thời gian: 270 phút

Bài 1 (7 điểm)
Cho đường tròn $(O)$ và 2 điểm cố định $B,C$ trên đường tròn sao cho $BC$ không là đường kính của $(O)$, $A$ là một điểm di động trên đường tròn, $A$ không trùng với $B,C$. Gọi $D,K,J$ lần lượt là trung điểm của $BC,CA,AB$ và $E,M,N$ lần lượt là hình chiếu vuông góc của $A,B,C$ trên $BC, DJ, DK$. Các tiếp tuyến tại $M,N$ của đường tròn ngoại tiếp tam giác $EMN$ cắt nhau tại $T$. Chứng minh $T$ là điểm cố định.

Bài 2 (7 điểm)
Trên một cánh đồng hình chữ nhật kích thước $m\times n$ ô vuông gồm $m$ hàng và $n$ cột người ta đặt một số máy bơm nước vào các ô vuông. Biết rằng mỗi máy bơm nước có thể tưới nước cho các ô vuông có chung cạnh với nó và các ô vuông cùng cột với nó và cách nó đúng một ô vuông . Tìm số nhỏ nhất các máy bơm nước sao cho các máy bơm nước có thể tưới hết cả cánh đồng trong 2 trường hợp:
a) $m=4$
b) $m=3$

Bài 3 (7 điểm)
Cho số nguyên tố $p\ge 17$. Chứng minh rằng $t=3$ là số nguyên dương lớn nhất thỏa mãn điều kiện: Với các số nguyên bất kì $a,b,c,d$ sao cho $abc$ không chia hết cho $p$ và $a+b+c$ chia hết cho $p$ thì tồn tại các số nguyên $x,y,z$ thuộc tập $\{0;1;...;\left[\frac{p}{t}\right]-1\}$ sao cho $ax+by+cz+d\vdots p$



Ngày thi thứ 2

Bài 4: (7 điểm)
Cho dãy số $(x_n)$ xác định bởi $x_1=1,x_2=2011$ và $x_{n+2}=4022x_{n+1}-x_n,\forall n\in \mathbb N$.
Chứng minh rằng $\frac{x_{2012}+1}{2012}$ là số chính phương.

Bài 5: (7 điểm)
Chứng minh rằng $c=10\sqrt{24}$ là hằng số lớn nhất thỏa mãn điều kiện: nếu có các số dương $a_1,a_2,...a_{17}$ sao cho: $$\sum_{i=1}^{17}{a_i^2}=24\qquad ;\qquad \sum_{i=1}^{17}{a_i^3}+\sum_{i=1}^{17}{a_i}<c$$ Thì với mọi $i,j,k$ thỏa mãn $1\le i<j<k\le 17$, ta luôn có $a_i,a_j,a_k$ là độ dài ba cạnh của một tam giác.

Bài 6: (7 điểm)
Có 42 học sinh tham dự kì thi chọn đội tuyển Olympic toán quốc tế. Biết rằng một học sinh bất kì quen đúng 20 học sinh khác. Chứng minh rằng ta có thể chia 42 học sinh thành 2 nhóm hoặc 21 nhóm sao cho số học sinh trong các nhóm bằng nhau và 2 học sinh bất kì trong cùng nhóm thì quen nhau.

(Cuối giờ ban tổ chức đã thu lại hết đề nên đây chỉ là đề mình ghi lại, không phải nguyên văn :wacko: )

Bài viết đã được chỉnh sửa nội dung bởi hoangduc: 17-04-2012 - 16:57

----------------------------------------------------

HỌC, HỌC NỮA, HỌC MÃI

#3
PTH_Thái Hà

PTH_Thái Hà

    David Tennant -- Doctor Who

  • Thành viên
  • 522 Bài viết
ngu hình từ bé nên bài hình bỏ, bài số thì ko chém được, bài tổ hợp thì lí luận suông
rút cục chả làm được gì
ngày mai chắc cũng thế nốt

hình như có mỗi bạn ở Cần Thơ, ngày mai làm quen phát

Bài viết đã được chỉnh sửa nội dung bởi PTH_Thái Hà: 16-04-2012 - 16:54

Giải nhì quốc gia. Yeah

#4
Karl Heinrich Marx

Karl Heinrich Marx

    Sĩ quan

  • Thành viên
  • 321 Bài viết
Mình nhìn qua không biết lập luận đúng câu tổ hợp không nữa.
kq câu a là n máy bơm.
Câu b đáp số của mình nếu $n=4k$ thì số máy bơm cần có là $3k+1$ còn nếu $n \ne 4k$ thì số máy bơm tối thiểu là $n-\lfloor \frac{n}{4} \rfloor$

Bài viết đã được chỉnh sửa nội dung bởi Karl Heinrich Marx: 16-04-2012 - 23:06


#5
The Gunner

The Gunner

    Hạ sĩ

  • Thành viên
  • 93 Bài viết
câu a hình như bạn ko đúng thì phải. Ý tưởng của mình là thế này ko bik đúng ko. Ta tô theo hình zich zắc. Theo kiểu đường chéo của hình vuông 3x3 thì thấy. Một vòi nước chỉ có thể tưới nhiều nhất 2 ô trong số các ô ta đã tô. Ví dụ với n=7 thì ta cần 4 vòi nước. Mình biện luận ra với n=6k+1 thì có ít nhất T=3k+1, với n=6k+2 thì có ít nhất T+1, n=6k+3 và 6k+4 thì có đáp số là T+2, n=6k+5 và 6k+6 thì có T+3. Còn với câu b mình có đáp số là kq câu a cộng thêm với $[\frac{n}{6}+1]$

Những ngày cuối cùng còn học toán

winwave1995

#6
Karl Heinrich Marx

Karl Heinrich Marx

    Sĩ quan

  • Thành viên
  • 321 Bài viết

câu a hình như bạn ko đúng thì phải. Ý tưởng của mình là thế này ko bik đúng ko. Ta tô theo hình zich zắc. Theo kiểu đường chéo của hình vuông 3x3 thì thấy. Một vòi nước chỉ có thể tưới nhiều nhất 2 ô trong số các ô ta đã tô. Ví dụ với n=7 thì ta cần 4 vòi nước. Mình biện luận ra với n=6k+1 thì có ít nhất T=3k+1, với n=6k+2 thì có ít nhất T+1, n=6k+3 và 6k+4 thì có đáp số là T+2, n=6k+5 và 6k+6 thì có T+3. Còn với câu b mình có đáp số là kq câu a cộng thêm với $[\frac{n}{6}+1]$

Bạn có thể minh họa hình vẽ được không

#7
The Gunner

The Gunner

    Hạ sĩ

  • Thành viên
  • 93 Bài viết
sr, hiện tại máy mình ko thể minh họa đc. Mình minh họa thế này nhé. Đặt tọa độ điểm. Ta sẽ tô các ô (1,1);(2,2);(3,3),(2,4);(1,5) rồi cứ tiếp tục như thế mà cách tô này hình như chưa tối ưu nên phải biện luận :) mình cũng ko chắc lắm

Những ngày cuối cùng còn học toán

winwave1995

#8
Karl Heinrich Marx

Karl Heinrich Marx

    Sĩ quan

  • Thành viên
  • 321 Bài viết
Thế chẳng phải mỗi cột có 1 ô còn j` thế thì là n cái chứ là mấy hả bạn :|

#9
nguyen thai phuc

nguyen thai phuc

    Sĩ quan

  • Thành viên
  • 430 Bài viết
\[{\rm{ax}} + by + cz = x(a + b + c) + b(y - x) + c(z - x)\]
đặt
\[\left[ {\frac{p}{t}} \right] - 1 = k;y - x = u;z - x = v\]
Với t>3 thì cho b=c ,ta cần chứng minh b(m+n) tạo thành hệ thặng dư module p
Mặt khác m+n chạy từ -2k đến 2k, tức là có 4k+1 số m+n module p => 4k+1 số b(m+n) module p
mà với t>3 thì 4k+1<p, vô lý.
với t=3
thì ta cần c/m luôn tồn tại m;n thuộc khoảng [-k;k] để
\[bu + cv \equiv d\left( {\bmod p} \right)\forall d \in \left[ {0;p - 1} \right]\]
Lại có u thuộc khoảng [-k;k] nên có 2k+1 số u, tức là có 2k+1 số bu khác nhau module p
=> có 2k+1 số d-bu khác nhau module p,
và có 2k+1 số cv khác nhau module p
mặt khác 4k+2>p với t=3;p>13 nên sẽ tồn tại 2 số bằng nhau (dirichle), tức là d-bu=cv hay d=bu+cv(modulo p)
ta có điều phải chứng minh
Hình đã gửi

#10
The Gunner

The Gunner

    Hạ sĩ

  • Thành viên
  • 93 Bài viết
sr, mình đọc ko kĩ đề. Tại mình giải trường hợp n hàng và m cột :)

Những ngày cuối cùng còn học toán

winwave1995

#11
alex_hoang

alex_hoang

    Thượng úy

  • Hiệp sỹ
  • 1152 Bài viết
Mình cũng thuộc dạng cực kì ngu hình may sao qua MS thấy lời giải này giới thiêuj với mọi người
Kí hiệu $(XY)$ chỉ đường tròn đường kính $(XY).$
Gọi$M', N'$theo thứ tự là hình chiếu của $B, C $trên $AC, AB. H$ là trực tâm tam giác $ABC$ Ta có các bộ điểm thẳng hàng $(B, M, H, M') (C, N, H, N').$Dễ thấy$ D, M, H, N $đồng viên.
$X$ là hình chiếu của $H$trên $AD.$ Khi đó,$X$ thuộc $(HA), (HD).$
Xét 3 đường tròn $(BC), (HA), (HD)$ có các trục đẳng phương $M'N', XH, BC $đồng quy tại $I.$
Chú ý rẳng $AE, BM', CN' $đồng quy do đó $(BCDI)= -1$. Qua phép chiếu xuyên tâm H ta có $H(MNDX)=-1$nên tứ giác $MDNX$điều hòa suy ra$ X, E, T$thẳng hàng.
Gọi $S$ là giao điểm của$OD$và$(HD)$ ta có $HS$ song song với $BC$và $D$ là trung điểm $BC$ suy ra $H(SDBC)=-1$ hay $H(SDMN)=-1$. Do đó $DMSN$điều hòa suy ra $S, T, D$thẳng hàng.
Dề thấy (bằng cộng góc)$ DM' $là tiếp tuyến của $(HA)$ dó đó $DC^2=DM'^2=DX.DA.$
Các tam giác$AEX $và $ADH$đồng dạng nên ta có $\frac{AE}{XA}=\frac{AD}{HA}$Ta có $TD$song song với$AH.$Vì thế theo định lý Thales ta có $\frac{DT}{DX}=\frac{AE}{AX}$ suy ra [$DT=DX.\frac{AE}{AX}=DX.\frac{DA}{AH}=\frac{DA.DX}{AH}=\frac{DC^2}{2OD}$không đổi.
Vậy$T$ là điểm cố định

Bài viết đã được chỉnh sửa nội dung bởi alex_hoang: 17-04-2012 - 00:57

alex_hoang


HẸN NGÀY TRỞ LẠI VMF THÂN MẾN

http://www.scribd.co...oi-Ban-Cung-The

#12
perfectstrong

perfectstrong

    $LOVE(x)|_{x =\alpha}^\Omega=+\infty$

  • Quản lý Toán Ứng dụng
  • 4996 Bài viết

\[{\rm{ax}} + by + cz = x(a + b + c) + b(y - x) + c(z - x)\]
đặt
\[\left[ {\frac{p}{t}} \right] - 1 = k;y - x = u;z - x = v\]
Với t>3 thì cho b=c ,ta cần chứng minh b(m+n) tạo thành hệ thặng dư module p
Mặt khác m+n chạy từ -2k đến 2k, tức là có 4k+1 số m+n module p => 4k+1 số b(m+n) module p
mà với t>3 thì 4k+1<p, vô lý.
với t=3
thì ta cần c/m luôn tồn tại m;n thuộc khoảng [-k;k] để
\[bu + cv \equiv d\left( {\bmod p} \right)\forall d \in \left[ {0;p - 1} \right]\]
Lại có u thuộc khoảng [-k;k] nên có 2k+1 số u, tức là có 2k+1 số bu khác nhau module p
=> có 2k+1 số d-bu khác nhau module p,
và có 2k+1 số cv khác nhau module p
mặt khác 4k+2>p với t=3;p>13 nên sẽ tồn tại 2 số bằng nhau (dirichle), tức là d-bu=cv hay d=bu+cv(modulo p)
ta có điều phải chứng minh

Anh ơi, $m,n$ là gì trong bài này hả anh?
Luôn yêu để sống, luôn sống để học toán, luôn học toán để yêu!!! :D
$$\text{LOVE}\left( x \right)|_{x = \alpha}^\Omega = + \infty $$
I'm still there everywhere.

#13
perfectstrong

perfectstrong

    $LOVE(x)|_{x =\alpha}^\Omega=+\infty$

  • Quản lý Toán Ứng dụng
  • 4996 Bài viết

Kí hiệu $(XY)$ chỉ đường tròn đường kính $(XY).$
Hình đã gửi
Gọi $M', N'$ theo thứ tự là hình chiếu của $B, C $trên $AC, AB$. $H$ là trực tâm $\vartriangle ABC$
Ta có các bộ điểm thẳng hàng $(B, M, H, N') (C, N, H, M').$Dễ thấy$ D, M, H, N $đồng viên.
$X$ là hình chiếu của $H$ trên $AD$. Khi đó,$X$ thuộc $(HA), (HD).$
Xét 3 đường tròn $(BC), (HA), (HD)$ có các trục đẳng phương $M'N', XH, BC $đồng quy tại $I$.
Chú ý rẳng $AE, BM', CN' $đồng quy do đó $(BCEI)= -1$. Qua phép chiếu xuyên tâm H ta có $H(MNEX)=-1$nên tứ giác $MENX$điều hòa suy ra$ X, E, T$thẳng hàng.
Gọi $S$ là giao điểm của $OD$và $(HD)$ ta có $HS \parallel BC$và $D$ là trung điểm $BC$ suy ra $H(SDBC)=-1$ hay $H(SDMN)=-1$. Do đó $DMSN$điều hòa suy ra $S, T, D$thẳng hàng.
Dề thấy (bằng cộng góc) $ DM' $ là tiếp tuyến của $(HA)$ dó đó $DC^2=DM'^2=DX.DA.$
$\vartriangle AEX \sim \vartriangle ADH \Rightarrow \frac{AE}{XA}=\frac{AD}{HA}$
Ta có $TD \parallel AH$. Vì thế theo định lý Thales ta có $\frac{DT}{DX}=\frac{AE}{AX}$
$\Rightarrow DT=DX.\frac{AE}{AX}=DX.\frac{DA}{AH}=\frac{DA.DX}{AH}=\frac{DC^2}{2OD}$không đổi.
Vậy$T$ là điểm cố định

Em sửa lại chút (theo ý em là đúng) và up cái hình. :D
Luôn yêu để sống, luôn sống để học toán, luôn học toán để yêu!!! :D
$$\text{LOVE}\left( x \right)|_{x = \alpha}^\Omega = + \infty $$
I'm still there everywhere.

#14
tungc3sp

tungc3sp

    Binh nhất

  • Thành viên
  • 46 Bài viết
Sao mà lại phải thu lại đề nhỉ?

alex_hoang:hỏi chúa bạn ơi >:)

Bài viết đã được chỉnh sửa nội dung bởi alex_hoang: 17-04-2012 - 18:48

tungk45csp

#15
toanhocmuonmau

toanhocmuonmau

    Sĩ quan

  • Thành viên
  • 438 Bài viết
Bài bất đẳng thức mình thấy hay đấy. :D

Đặt $a_i=\sqrt{24}x_i,$ khi đó yêu cầu bài toán tương đương với: Chứng minh rằng $C=10$ là hằng số lớn nhất sao cho nếu có $17$ số thực dương $x_1,\,x_2,\, \ldots,\, x_{17}$ thỏa mãn $$x_1^2+x_2^2+\cdots +x_{17}^2=1$$ và $$24(x_1^3+x_2^3+\cdots +x_{17}^3)+(x_1+x_2+\cdots +x_{17}) <C$$ thì với mọi $i,\, j,\, k$ thỏa mãn $1 \le i <j<k\le 17,$ ta có $x_i,\, x_j,\, x_k$ là độ dài ba cạnh của một tam giác.

Trước hết, ta sẽ chứng minh yêu cầu bài toán thỏa với $C=10.$ Không mất tính tổng quát, ta chỉ cần chứng minh $x_1,\,x_2,\, x_3$ là độ dài ba cạnh của một tam giác. Để ý rằng với mọi $0<t<1,$ ta có $$24t^3+t -(16t^4+9t^2)=t(1-t)(4t-1)^2 \ge 0.$$ Do đó, từ giả thiết ta suy ra $$16(x_1^4+x_2^4+\cdots +x_{17}^4)+9(x_1^2+x_2^2+\cdots +x_{17}^2) <10,$$ hay $$16(x_1^4+x_2^4+\cdots +x_{17}^4)<1=(x_1^2+x_2^2+\cdots +x_{17}^2)^2.$$ Bây giờ, sử dụng bất đẳng thức Cauchy-Schwarz, ta có $$16(x_1^4+x_2^4+\cdots +x_{17}^4)=\left(2+\underset{14 \text{ số}}{\underbrace{1+1+\cdots +1}}\right)\left[(x_1^4+x_2^4+x_3^4)+(x_4^4+x_5^4+\cdots +x_{17}^4)\right] \ge \left[ \sqrt{2(x_1^4+x_2^4+x_3^4)}+x_4^2+x_5^2+\cdots +x_{17}^2\right]^2.$$ Do đó, kết hợp với trên, ta thu được $$\sqrt{2(x_1^4+x_2^4+x_3^4)} <x_1^2+x_2^2+x_3^2,$$ hay $$2(x_1^4+x_2^4+x_3^4)<(x_1^2+x_2^2+x_3^2)^2.$$ Mặt khác, ta có đồng nhất thức $$\begin{aligned} (x_1^2+x_2^2+x_3^2)^2&- 2(x_1^4+x_2^4+x_3^4)= \\ &=2(x_1+x_2+x_3)(x_1+x_2-x_3)(x_2+x_3-x_1)(x_3+x_1-x_2). \end{aligned} $$ Do vậy, kết hợp với bất đẳng thức ở trên, ta suy ra $x_1,\,x_2,\,x_3$ là độ dài ba cạnh của một tam giác.

Phần chứng minh $10$ là hằng số lớn nhất khá dễ nên xin dành lại cho bà con. Giờ mình phải đi dạy tiếp đây. :D

Bài viết đã được chỉnh sửa nội dung bởi toanhocmuonmau: 17-04-2012 - 19:48

The love makes us stronger!

V. Q. B. Can


#16
NguyThang khtn

NguyThang khtn

    Thượng úy

  • Hiệp sỹ
  • 1468 Bài viết
Lần đầu em đọc bài này em cũng chưa có ý tưởng gì nhiều!
Cho em hỏi hướng tư duy bào này được không?

It is difficult to say what is impossible, for the dream of yesterday is the hope of today and the reality of tomorrow

 


#17
alex_hoang

alex_hoang

    Thượng úy

  • Hiệp sỹ
  • 1152 Bài viết
Lâu quá các thành viên của $VMF$ mới thấy anh Cẩn ghé thăm diễn đàn :icon6:
Mình thử chém thử bài 1 ngày hai coi sao
Trước hết ta xét phương trình
$$x^2-4022x+1=0$$
Phương trình này có hai nghiệm phân biệt là $x=2011 \pm \sqrt {{{2011}^2} - 1} $
Vậy ta tìm được công thức tổng quát cho dãy số trên là
\[{x_n} = \frac{1}{2}\left[ {{{\left( {2011 + \sqrt {{{2011}^2} - 1} } \right)}^{n - 1}} + {{\left( {2011 - \sqrt {{{2011}^2} - 1} } \right)}^{n - 1}}} \right]\]
Mà ta chú ý rằng
\[2011 \pm \sqrt {{{2011}^2} - 1} = {\left( {\sqrt {1006} \pm \sqrt {1005} } \right)^2}\]
Từ đây ta có
\[\frac{{{x_n} + 1}}{{2012}} = \frac{1}{{2.2012}}{\left[ {{{\left( {\sqrt {1006} + \sqrt {1005} } \right)}^{n - 1}} + {{\left( {\sqrt {1006} - \sqrt {1005} } \right)}^{n - 1}}} \right]^2}\]
Vậy thì
\[\frac{{{x_n} + 1}}{{2012}} = {\left[ {\frac{{{{\left( {\sqrt {1006} + \sqrt {1005} } \right)}^{n - 1}} + {{\left( {\sqrt {1006} - \sqrt {1005} } \right)}^{n - 1}}}}{{2\sqrt {1006} }}} \right]^2}\]
Ta dễ chứng minh được biểu thức
\[\frac{{{{\left( {\sqrt {1006} + \sqrt {1005} } \right)}^{n - 1}} + {{\left( {\sqrt {1006} - \sqrt {1005} } \right)}^{n - 1}}}}{{2\sqrt {1006} }}\]
Đạt giá trị nguyên khi và chỉ khi $n$ đạt giá trị chẵn và $\ge 2$
Vậy thì nó cũng đúng với $n=2012$
Mình không dự đoán trước được dãy nên làm cách củ chuối này mong mọi người thông cảm vì khả năng có hạn
alex_hoang


HẸN NGÀY TRỞ LẠI VMF THÂN MẾN

http://www.scribd.co...oi-Ban-Cung-The

#18
alex_hoang

alex_hoang

    Thượng úy

  • Hiệp sỹ
  • 1152 Bài viết
Bên MS họ đưa ra bài toán tổng quát hơn và lời giải cũng hay hơn.Mình trích lại cho thành viên diễn đàn ta tham khảo vậy.

Bài 4 mình đang thử chứng minh bài toán tổng quát hơn là:

Cho p là số nguyên tố lẻ. Xét dãy số $(x_n)$xác định bởi:

$$x_1=1,x_2=p,x_{n+2}=2px_{n+1}-x_n, n \ge 1$$

Chứng minh rằng $\frac{x_{p+1}+1}{p+1}$ là số chính phương.

Thử với $p=3,5$ thì đúng rồi.


Em thử xem nào: $x_{n+2}=2p(2px_n - x_{n-1})-x_n=(4p^2-2)x_n-x_{n-2}$
Xét dãy $(y_n): y_1=1;y_2=2p-1; y_{n+1}=2py_n-y_{n-1}$
Ta chứng minh bằng quy nạp rằng $(p+1)(y_n)^2+1=x_{2n}$
Thật vậy, từ cách cho $y_n$ có
$y_n^2-y_{n-1}y_{n+1}=2-2p$
$(y_{n+1}+y_{n-1})^2=4p^2y_n^2\Rightarrow y_{n+1}^2=4p^2y_n^2-y_{n-1}^2-2y_{n-1}y_{n+1}=(4p^2-2)y_n^2-y_{n-1}^2 -2(2p-2)\Rightarrow y_{n+1}=\frac{(4p^2-2)(x_{2n}+1)}{p+1}-\frac{x_{2n-2}+1}{p+1}+2(2-2p)=\frac{x_{2n+2}+1}{p+1}$
Suy ra $\frac{x_{2n}+1}{p+1}$ chính phương với mọi n, p lẻ nên có đpcm.


alex_hoang


HẸN NGÀY TRỞ LẠI VMF THÂN MẾN

http://www.scribd.co...oi-Ban-Cung-The

#19
The Gunner

The Gunner

    Hạ sĩ

  • Thành viên
  • 93 Bài viết
Bài 6: Giả sử ko có cách chia 2 nhóm. Ban đầu ta chia các thí sinh đó thành các nhóm gồm hai thành viên và hai thí sinh đó phải quen nhau. Gọi tập các nhóm là S. Nếu S có đủ 21 nhóm thì ta đc đpcm. Xét trường hợp S có ít hơn 21 nhóm và các thí sinh ko thuộc S ko thể tạo nên1 cặp vs thí sinh khác để thuộc S. Thì ta xét thí sinh b thuộc S quen với a ko thuộc S mà người thuộc nhóm vs b trong S là c cũng quen vs d ko thuộc S thì ta chỉ cần cho a và b, c và d thành 1 cặp. Cách chọn này luôn tồn tại vì số người quen cuả mỗi người là 20 mà số nhóm trong S nhỏ hơn hoặc bằng 20. Như thế sau mỗi lần thực hiện số người ko thuộc S giảm đi 2. Mà số người ko thuộc S ko âm nên qúa trình này phải dừng. Vì ta giả sử ko có cách chia 2 nhóm tức là tập đỉnh của đồ thị này ko thể chia làm 2 thành phần đày đủ 21 phần và ko có cầu, nên quá trình này ko thể dừng ở 2.Vì nếu nó dừng ở 2 thì do trong S có 20 cặp. Bây giờ ta cần chứng minh quá trình dừng ở 0. Theo điều giả sử thì suy ra đồ thị G ( là 42 đỉnh ban đầu) là một đồ thị liên thông, thật vậy, giả sử G ko liên thông thì G chỉ có thể có 2 thành phầ liên thông vì bậc của mỗi đỉnh đúng bằng 20, và khi đó mỗi thành phần liên thông sẽ là $K_{21}$ trái với điều giả sử. Do đó G liên thông, nếu quá trình này dừng đến lúc chỉ còn 2 điểm ko thuộc S, và ko thể tiến hành được nữa thì giả sử đó là hai điểm $x$ và $y$ thì ta có đường đi từ $x$ đến $y$. xét các đường đi từ $x$ đến $y$ thì ta có:gọi X và Y lần lượt là tập các láng giềng của $x$ và $y$. Khi đó $|X|=|Y|=20$. Ta chỉ cần chứng minh tồn tại đường đi có số chẵn đỉnh thì khi đó ta được các cặp thỏa mãn. xét các đường đi từ x đến y đi qua các cặp thuộc S thì giả sử nó đi qua hai điểm A,B là một cặp thuộc S thì nếu A thuộc X, và B thuộc Y thì ta có đpcm. Nếu ko có cặp AB nào mà A thuộc X và B thuộc Y như trên. THì ta xét một cầu nối giữa hai thành phần liên thông của X và Y thì đầu mút của hai cầu đó là C,D thì ta cũng được đường x,C,D,y là một đường đi chẵn đỉnh. Vậy ta luôn có thể chọn được 21 cặp thỏa mãn. (đpcm)
P/s: đã edit

Bài viết đã được chỉnh sửa nội dung bởi The Gunner: 19-04-2012 - 00:33

Những ngày cuối cùng còn học toán

winwave1995

#20
alex_hoang

alex_hoang

    Thượng úy

  • Hiệp sỹ
  • 1152 Bài viết
Đây là đáp án và bình luận các bạn nhé :)

File gửi kèm


alex_hoang


HẸN NGÀY TRỞ LẠI VMF THÂN MẾN

http://www.scribd.co...oi-Ban-Cung-The




0 người đang xem chủ đề

0 thành viên, 0 khách, 0 thành viên ẩn danh